前往小程序,Get更优阅读体验!
立即前往
首页
学习
活动
专区
工具
TVP
发布
社区首页 >专栏 >矩阵分析笔记(六)矩阵等价与线性映射的最简表示

矩阵分析笔记(六)矩阵等价与线性映射的最简表示

作者头像
mathor
发布2020-10-15 10:34:59
1.6K0
发布2020-10-15 10:34:59
举报
文章被收录于专栏:mathor
矩阵等价

矩阵A\cong B的充分必要条件是存在m阶可逆矩阵Pn阶可逆矩阵Q,使PAQ=B


线性映射的最简表示

在指定了空间V_1V_2的基之后,便可以求得线性映射\mathscr{A}:V_1\to V_2在指定一对基下的矩阵表示。但是空间基是不唯一的,自然应该考虑以下两个问题:

  1. 线性映射在不同对基下的矩阵表示之间有什么关系?
  2. 对一个线性映射,能否选择一对基,使它的矩阵表示最简单(零多)?

先回答第一个问题

\mathscr{A}V_1\to V_2的一个线性映射,\alpha_1,\alpha_2,...,\alpha_n\alpha^{'}_1,\alpha^{'}_2,...,\alpha^{'}_nV_1的两组基,由\alpha_i\alpha^{'}_i的过渡矩阵为P。设\beta_1,\beta_2,...,\beta_m\beta^{'}_1,\beta^{'}_2,...,\beta^{'}_mV_2的两组基,由\beta_j\beta^{'}_j的过渡矩阵为Q。线性映射\mathscr{A}在基\alpha_1,\alpha_2,...,\alpha_n\beta_1,\beta_2,...,\beta_m下的矩阵表示为A,在基\alpha^{'}_1,\alpha^{'}_2,...,\alpha^{'}_n\beta^{'}_1,\beta^{'}_2,...,\beta^{'}_m下的矩阵表示为B,则

B = Q^{-1}AP

证明:

由假设条件知

$$ \begin{gather} \mathscr{A}(\alpha_1,\alpha_2,...,\alpha_n)=(\beta_1,\beta_2,...,\beta_m)A \tag{1}\\ \mathscr{A}(\alpha^{'}_1,\alpha^{'}_2,...,\alpha^{'}_n)=(\beta^{'}_1,\beta^{'}_2,...,\beta^{'}_m)B \tag{2}\\ (\alpha^{'}_1,\alpha^{'}_2,...,\alpha^{'}_n)=(\alpha_1,\alpha_2,...,\alpha_n)P \tag{3}\\ (\beta^{'}_1,\beta^{'}_2,...,\beta^{'}_m)=(\beta_1,\beta_2,...,\beta_m)Q \tag{4} \end{gather} $$

将式(3)和式(4)带入式(2)

$$ \begin{gather} \mathscr{A}(\alpha_1,\alpha_2,...,\alpha_n)P=(\beta_1,\beta_2,..,\beta_m)QB \tag{5} \end{gather} $$

将式(1)带入式(5)

$$ \begin{aligned} (\beta_1,\beta_2,...,\beta_m)AP=(\beta_1,\beta_2,...,\beta_m)QB \end{aligned} $$

因为(\beta_1,\beta_2,...,\beta_m)线性无关,故

AP=QB

由于Q是满秩方阵(因为过渡矩阵都是满秩的),所以

B = Q^{-1}AP \tag{6}

回答第二个问题

大学线性代数中有这么一个结论:对于m\times n矩阵A,总可经过初等变换(行变换和列变换)把它化为标准形

Q^{-1}AP=\begin{bmatrix}E_r& 0_{r\times (n-r)}\\0_{(m-r)\times r}&0_{(m-r)\times (n-r)}\end{bmatrix}_{m\times n},\ r = rank(A)

将式(6)带入式(2)

$$ \begin{aligned} \mathscr{A}(\alpha^{'}_1,\alpha^{'}_2,...,\alpha^{'}_n)&=(\beta^{'}_1,\beta^{'}_2,...,\beta^{'}_m)Q^{-1}AP\\ &=(\beta^{'}_1,\beta^{'}_2,...,\beta^{'}_m)\begin{bmatrix}E_r& 0_{r\times (n-r)}\\0_{(m-r)\times r}&0_{(m-r)\times (n-r)}\end{bmatrix}_{m\times n} \end{aligned} $$

所以,对于一个线性映射,一定可以找到一对基,使得线性映射对应的矩阵最简单


线性变换

接下来的线性映射\mathscr{A}都是指线性空间VV的映射,特称这样的\mathscr{A}为线性空间V的线性变换。由于线性变换时线性空间V到它自身的映射,所以只需取V的一组基\alpha_1,\alpha_2,...,\alpha_n即可

\alpha = \begin{bmatrix}\alpha_1,\alpha_2,...,\alpha_n\end{bmatrix}\begin{bmatrix}x_1\\x_2\\ \vdots x_n\end{bmatrix} \in V,若

\mathscr{A}(\alpha) = \begin{bmatrix}\alpha_1, \alpha_2,...,\alpha_n\end{bmatrix}\begin{bmatrix}y_1\\ y_2 \\ \vdots \\ y_n\end{bmatrix}

则原像\alpha与像\mathscr{A}(\alpha)的坐标变换公式为

\begin{bmatrix}y_1\\y_2\\ \vdots \\ y_n\end{bmatrix}=A\begin{bmatrix}x_1 \\ x_2 \\ \vdots \\ x_n\end{bmatrix} \tag{7}
例1

\mathbb{R}^3中线性变换\mathscr{A}将基

\alpha_1 = \begin{bmatrix}1\\1\\-1\end{bmatrix}, \alpha_2=\begin{bmatrix}0\\2\\-1\end{bmatrix},\alpha_3=\begin{bmatrix}1\\0\\-1\end{bmatrix}

变为基

\alpha^{'}_1 = \begin{bmatrix}1\\-1\\0\end{bmatrix}, \alpha^{'}_2=\begin{bmatrix}0\\1\\-1\end{bmatrix},\alpha^{'}_3=\begin{bmatrix}0\\3\\-2\end{bmatrix}
  1. \mathscr{A}在基\alpha_1,\alpha_2,\alpha_3下的矩阵表示A
  2. 求向量\xi=(1,2,3)^T\mathscr{A}(\xi)在基\alpha_1, \alpha_2,\alpha_3下的坐标
  3. 求向量\xi\mathscr{A}(\xi)在基\alpha^{'}_1,\alpha^{'}_2,\alpha^{'}_3下的坐标

解:(1)

$$ \because \mathscr{A}(\alpha_1,\alpha_2,\alpha_3)=(\alpha_1,\alpha_2,\alpha_3)A=(\alpha^{'}_1,\alpha^{'}_2,\alpha^{'}_3)\\ \therefore A = (\alpha_1,\alpha_2,\alpha_3)^{-1}(\alpha^{'}_1,\alpha^{'}_2,\alpha^{'}_3)=\begin{bmatrix}1&-1&-1\\-1&1&2\\0&1&1\end{bmatrix} $$

(2)

\xi=\begin{bmatrix}\alpha_1,\alpha_2,\alpha_3\end{bmatrix}\begin{bmatrix}k_1\\k_2\\k_3\end{bmatrix},即

\begin{bmatrix}1\\2\\3\end{bmatrix}=\begin{bmatrix}1&0&1\\1&2&0\\-1&-1&-1\end{bmatrix}\begin{bmatrix}k_1\\k_2\\k_3\end{bmatrix}

解得

k_1 = 10, k_2=-4,k_3=-9

所以\xi在基\alpha_1,\alpha_2,\alpha_3下的坐标为(10,-4,-9)^T

\mathscr{A}(\xi)在基\alpha_1,\alpha_2,\alpha_3下的坐标可由公式(7)

\begin{bmatrix}y_1\\y_2\\y_3\end{bmatrix}=\begin{bmatrix}1&-1&-1\\-1&1&2\\0&1&1\end{bmatrix}\begin{bmatrix}10\\-4\\-9\end{bmatrix}=\begin{bmatrix}23\\-32\\-13\end{bmatrix}

(3)

\xi=[\alpha_1,\alpha_2,\alpha_3]\begin{bmatrix}x_1\\x_2\\x_3\end{bmatrix},即

\begin{bmatrix}1\\2\\3\end{bmatrix}=\begin{bmatrix}1&0&0\\-1&1&3\\0&-1&-2\end{bmatrix}\begin{bmatrix}x_1\\x_2\\x_3\end{bmatrix}\Rightarrow \begin{bmatrix}x_1\\x_2\\x_3\end{bmatrix}=\begin{bmatrix}1\\-15\\6\end{bmatrix}

\mathscr{A}(\xi)在基\alpha_1,\alpha_2,\alpha_3下的坐标可由公式(7)

\begin{bmatrix}y_1\\y_2\\y_3\end{bmatrix}=\begin{bmatrix}1&-1&-1\\-1&1&2\\0&1&1\end{bmatrix}\begin{bmatrix}1\\-15\\6\end{bmatrix}=\begin{bmatrix}10\\-4\\-9\end{bmatrix}
例2

求线性空间\mathbb{R}^3绕指定了正方向的固定轴旋转角度\theta的变换\mathscr{A}的矩阵表示

解:以O为起点沿旋转轴正方向取单位长有向线段,记为e_z,再取以O为起点的另两单位长有向线段e_x,e_y,使得e_x,e_y,e_z构成线性空间V中的右手直角坐标系。入口基和出口基都选为e_x,e_y,e_z

\mathscr{A}(e_x,e_y,e_z)=\begin{bmatrix}e_x&e_y&e_z\end{bmatrix}\begin{bmatrix}cos\theta&-sin\theta&0\\sin\theta&cos\theta&0\\0&0&1\end{bmatrix}

因此\mathscr{A}的矩阵表示为\begin{bmatrix}cos\theta&-sin\theta&0\\sin\theta&cos\theta&0\\0&0&1\end{bmatrix}

例3

求几何空间中以XOY面为镜面反射变换\mathscr{B}的矩阵表示

解:

$$ \begin{aligned} \mathscr{B}(e_x,e_y,e_z)&=\begin{bmatrix}e_x&e_y&-e_z\end{bmatrix}\\ &=\begin{bmatrix}e_x&e_y&e_z\end{bmatrix}\begin{bmatrix}1&0&0\\0&1&0\\0&0&-1\end{bmatrix} \end{aligned} $$

因此\mathscr{B}的矩阵表示为\begin{bmatrix}1&0&0\\0&1&0\\0&0&-1\end{bmatrix}

线性变换的运算

\mathscr{A},\mathscr{B}是线性空间V的两个线性变换,\lambda \in \mathbb{F}

  1. 加法:(\mathscr{A}+\mathscr{B})(\alpha)=\mathscr{A}(\alpha)+\mathscr{B}(\alpha)
  2. 乘法:\mathscr{AB}(\alpha)=\mathscr{A}(\mathscr{B}(\alpha))
  3. 数乘:(\lambda\mathscr{A})(\alpha)=\lambda \mathscr{A}(\alpha)
  4. 可逆:设\mathscr{A}\mathscr{B}=\mathscr{B}\mathscr{A}=E,其中E为恒等变换,这时变换\mathscr{B}称为\mathscr{A}的拟变换,记为\mathscr{A}^{-1}

问题:对于一般的线性映射,能否定义加法、乘法、数乘?

很明显加法和数乘都可以,乘法不行(维度不匹配)


不同基下的矩阵关系

\mathscr{A}为线性空间V上的线性变换,设\alpha_1,\alpha_2,...,\alpha_n\alpha^{'}_1,\alpha^{'}_2,...,\alpha^{'}_nV的基且过渡矩阵为P。若\mathscr{A}在基\alpha_1,\alpha_2,...,\alpha_n下的矩阵表示为A,在基\alpha^{'}_1,\alpha^{'}_2,...,\alpha^{'}_n下的矩阵表示为B,则

B=P^{-1}AP

证明:

由已知得

$$ \begin{aligned} \mathscr{A}(\alpha_1,\alpha_2,...,\alpha_n)=(\alpha_1,\alpha_2,...,\alpha_n)A \\ \mathscr{A}(\alpha^{'}_1,\alpha^{'}_2,...,\alpha^{'}_n)=(\alpha^{'}_1,\alpha^{'}_2,...,\alpha^{'}_n)B \\ (\alpha^{'}_1,\alpha^{'}_2,...,\alpha^{'}_n)=(\alpha_1,\alpha_2,...,\alpha_n)P \end{aligned} $$

于是有

$$ \begin{aligned} \because (\alpha^{'}_1,\alpha^{'}_2,...,\alpha^{'}_n)&=(\alpha_1,\alpha_2,...,\alpha_n)P\\ \therefore \mathscr{A}(\alpha^{'}_1,\alpha^{'}_2,...,\alpha^{'}_n)&=\mathscr{A}(\alpha_1,\alpha_2,...,\alpha_n)P\\ &=(\alpha_1,\alpha_2,...,\alpha_n)AP\\ \because \mathscr{A}(\alpha^{'}_1,\alpha^{'}_2,...,\alpha^{'}_n)&=((\alpha_1,\alpha_2,...,\alpha_n)P)B\\ &=(\alpha_1,\alpha_2,...,\alpha_n)PB\\ \therefore AP=PB &\Rightarrow B=P^{-1}AP \end{aligned} $$

相似

A,B\in \mathbb{F}^{m\times n},若存在P\in \mathbb{F}^{n\times n},满足

B=P^{-1}AP

则称BA相似,记为B\sim A


同构

V_1,V_2为线性空间,若存在一一映射\sigma:V_1\to V_2满足\forall \alpha, \beta \in V_1, \lambda \in \mathbb{F},有

$$ \begin{aligned} \sigma(\alpha+\beta)&=\sigma(\alpha)+\sigma(\beta)\\ \sigma(\lambda \alpha)&=\lambda \sigma(\alpha) \end{aligned} $$

则称V_1V_2同构,\sigma称为同构映射

同构的充要条件

数域\mathbb{F}上两个有限维线性空间V_1,V_2同构的充要条件是\dim (V_1)=\dim (V_2)

同构的性质

同构映射具有以下四个基本性质

  1. \sigma(0)=0, \sigma(\alpha)=-\sigma(\alpha)
  2. \sigma(k_1\alpha_1+k_2\alpha_2+···+k_s\alpha_s)=k_1\sigma(\alpha_1)+k_2\sigma(\alpha_2)+···k_s\sigma(\alpha_s)
  3. V中向量组\alpha_1,\alpha_2,...,\alpha_s线性相(无)关\Longleftrightarrow\sigma(\alpha_1),\sigma(\alpha_2),...,\sigma(\alpha_s)线性相(无)关
  4. 如果V_1V的一个子空间,则V_1\sigma下的像集合\sigma(V_1)=\{\sigma(\alpha)\mid \alpha \in V_1\}\sigma(V)的子空间,并且V_1\sigma(V_1)维数相同
本文参与 腾讯云自媒体同步曝光计划,分享自作者个人站点/博客。
如有侵权请联系 cloudcommunity@tencent.com 删除

本文分享自 作者个人站点/博客 前往查看

如有侵权,请联系 cloudcommunity@tencent.com 删除。

本文参与 腾讯云自媒体同步曝光计划  ,欢迎热爱写作的你一起参与!

评论
登录后参与评论
0 条评论
热度
最新
推荐阅读
目录
  • 矩阵等价
  • 线性映射的最简表示
  • 线性变换
  • 例1
  • 例2
  • 例3
  • 线性变换的运算
  • 不同基下的矩阵关系
  • 相似
  • 同构
  • 同构的充要条件
  • 同构的性质
领券
问题归档专栏文章快讯文章归档关键词归档开发者手册归档开发者手册 Section 归档